آيا شما مايل به راه افتادن تاپيك ماراتن نامساوي مي باشي


  • مجموع رای دهندگان
    339

m-saghaei

New Member
ارسال ها
338
لایک ها
258
امتیاز
0
پاسخ : ماراتن نامساوي

من فقط اینو میدونم:

یعنی

اگه میشه دوستان یه راهنمایی کنن!
ما میخوایم حداکثر رو بیابیم نه حداقل رو !
بهتره اینی که میگید رو اینطوری بگیم مثلا:
 

REZA 73

Active Member
ارسال ها
139
لایک ها
184
امتیاز
43
پاسخ : ماراتن نامساوي

اولا واضحه که ماکزیمم وقتی به دست نمیاد که هر سه تا مثبت باشند.
یعنی حداقل یکیشون باید منفی باشه(وقتی هر سه تا هم منفی باشن مثه اینه که هر سه تا مثبت هستند)
وقتی c منفی باشه اونوقت ماکزیمم به دست میاد دلیلش هم واضحه چون قدر مطلق 2ac و 2bc به عبارت افزوده میشه:
حالا فرض میکیم m قرینه c هست:
که کوچکتر هست از:

که به وضوح از 19 کوچکتره.
در ضمن:

وقتی هم دو تا منفی باشند مثله حالتیه که یکی منفی باشه. امیدوارم واضح نوشته باشم.
 

Dadgarnia

New Member
ارسال ها
1,350
لایک ها
1,127
امتیاز
0
پاسخ : ماراتن نامساوي

اولا واضحه که ماکزیمم وقتی به دست نمیاد که هر سه تا مثبت باشند.
یعنی حداقل یکیشون باید منفی باشه(وقتی هر سه تا هم منفی باشن مثه اینه که هر سه تا مثبت هستند)
وقتی c منفی باشه اونوقت ماکزیمم به دست میاد دلیلش هم واضحه چون قدر مطلق 2ac و 2bc به عبارت افزوده میشه:
حالا فرض میکیم m قرینه c هست:
که کوچکتر هست از:

که به وضوح از 19 کوچکتره.
در ضمن:

وقتی هم دو تا منفی باشند مثله حالتیه که یکی منفی باشه. امیدوارم واضح نوشته باشم.
نمی دونم این استدلال تا چه حد درسته ولی اگه بیشتر توضیح بدین ممنون میشم.
 

REZA 73

Active Member
ارسال ها
139
لایک ها
184
امتیاز
43
پاسخ : ماراتن نامساوي

نمی دونم این استدلال تا چه حد درسته ولی اگه بیشتر توضیح بدین ممنون میشم.
که a و b مثبتن.بقیه هم دیگه سادس ازین استفاده شده که عبارت داخل قدر مطلق همه عبارت های سمت راست مثبت هستند.
 

sepidfekr

New Member
ارسال ها
711
لایک ها
637
امتیاز
0
پاسخ : ماراتن نامساوي

سوال بعد(امیدوارم تکراری نباشه):

 

m-saghaei

New Member
ارسال ها
338
لایک ها
258
امتیاز
0
پاسخ : ماراتن نامساوي

سوال بعد(امیدوارم تکراری نباشه):

میدونیم که:


پس:


که اگه سه تاشو باهم بگیریم درمیاد:


پس الان میخوایم بگیم:


که اگه کوشی بزنیم نتیجه میشه:


من سوال ندارم لطفا یکی لطف کنه بزاره!
 

sepidfekr

New Member
ارسال ها
711
لایک ها
637
امتیاز
0
پاسخ : ماراتن نامساوي

سوال بعد:

 

Dadgarnia

New Member
ارسال ها
1,350
لایک ها
1,127
امتیاز
0
پاسخ : ماراتن نامساوي

میدونیم که:


پس:


که اگه سه تاشو باهم بگیریم درمیاد:


پس الان میخوایم بگیم:


که اگه کوشی بزنیم نتیجه میشه:


من سوال ندارم لطفا یکی لطف کنه بزاره!
البته اون نامساوی اولی رو میشه با نامساوی مثلث هم نتیجه گرفت:
 

m-saghaei

New Member
ارسال ها
338
لایک ها
258
امتیاز
0
پاسخ : ماراتن نامساوي

میایم این تغییر متغیر رو انجام میدیم:


و اون فرضی هم که داده به خاطر اینه که الان نتیجه میشه


حالا مخرج مشترک میگیریم و بعدش در میاد که باید ثابت کنیم:


و اینجا از یه اتحاد مثلثاتی استفاده میکنیم که:


پس حالا باید ثابت کنیم که:


که اگه تغییر متغیر
رو انجام بدیم دقیقا میشه شور توی
 

sepidfekr

New Member
ارسال ها
711
لایک ها
637
امتیاز
0

sepidfekr

New Member
ارسال ها
711
لایک ها
637
امتیاز
0
پاسخ : ماراتن نامساوي

اون نامساوي جزئي فقط براي بزرگترين و كوچكترين ضلع درسته!
منظورت چیه؟اگه میشه یکم بیشتر توضیح بده!!!
من به آقای ارس هم گفتم ایشون مثال نقضی نتونستن پیدا کنن!!!
اگه میشه مثال نقض بیارید!!!
 

m-saghaei

New Member
ارسال ها
338
لایک ها
258
امتیاز
0
پاسخ : ماراتن نامساوي

منظورت چیه؟اگه میشه یکم بیشتر توضیح بده!!!
من به آقای ارس هم گفتم ایشون مثال نقضی نتونستن پیدا کنن!!!
اگه میشه مثال نقض بیارید!!!
مثلا 16 و 25 و 36 !
 

Dadgarnia

New Member
ارسال ها
1,350
لایک ها
1,127
امتیاز
0
پاسخ : ماراتن نامساوي

منظورت چیه؟اگه میشه یکم بیشتر توضیح بده!!!
من به آقای ارس هم گفتم ایشون مثال نقضی نتونستن پیدا کنن!!!
اگه میشه مثال نقض بیارید!!!
واضحه كه دو طرف مثبتن پس دو طرفو به توان دو مي رسونيم:
اين رو هم به توان دو مي رسونيم:


پس اگه
باشه نامساوي بالا برقرار نيست و اين توي هر مثلثي برقراره.
 

AHZolfaghari

Well-Known Member
ارسال ها
935
لایک ها
1,654
امتیاز
93
پاسخ : ماراتن نامساوي

a,b,c اضلاع یک مثلث هستند.نشان دهید:
با تغییر متغیر
داریم :



پس



با جمع زدن همگی این ها درنهایت باید ثابت کنیم که



که این هم بدیهی که چون نامساوی شور در حالت r=-2 می باشد !!!!
 

AHZolfaghari

Well-Known Member
ارسال ها
935
لایک ها
1,654
امتیاز
93
پاسخ : ماراتن نامساوي

سوال بعد :

اگر k,m,n سه عدد طبیعی باشند که دو به دو متمایزند اثبات کنید



 

m-saghaei

New Member
ارسال ها
338
لایک ها
258
امتیاز
0
پاسخ : ماراتن نامساوي

سوال بعد :

اگر k,m,n سه عدد طبیعی باشند که دو به دو متمایزند اثبات کنید



بالاخره حل شد!

اول بازش میکنیم میشه:


بعد با توجه به تقارن مسئله میتونیم فرض کنیم

بعد این تغییر متغیر رو انجام میدیم:
که در اون


بعدش اومدم اینارو جاگذاری کردم (حدودا یه ربع کامل داشتم مینوشتم!:4:) آخر آخرش شد این:



که اینم از صفر بیشتره!!!

(و خداوند خرکاری را آفرید!)
ولی مطمئنا راه کوتاه تری هم داره!
 

AHZolfaghari

Well-Known Member
ارسال ها
935
لایک ها
1,654
امتیاز
93
پاسخ : ماراتن نامساوي

بالاخره حل شد!

اول بازش میکنیم میشه:


بعد با توجه به تقارن مسئله میتونیم فرض کنیم

بعد این تغییر متغیر رو انجام میدیم:
که در اون


بعدش اومدم اینارو جاگذاری کردم (حدودا یه ربع کامل داشتم مینوشتم!:4:) آخر آخرش شد این:



که اینم از صفر بیشتره!!!

(و خداوند خرکاری را آفرید!)
ولی مطمئنا راه کوتاه تری هم داره!
راه حل من برای این سوال mixing هست .
اول k<n<m .
اول که ضرب می کنیم و همه رو میبریم سمت مثبت تساوی و اون عبارت رو
می گیریم.
حالا میشه ثابت کرد :

و حالا اثبات این که
نا منفی هستش هم سخت نیست

سوال بعد :
اگر x,y,z سه عدد حقیقی باشند که
اثبات کنید
 
بالا